Bạn chưa đăng nhập. Vui lòng đăng nhập để hỏi bài

Những câu hỏi liên quan
bé bông 2k9
Xem chi tiết
Akai Haruma
9 tháng 11 2021 lúc 21:27

Lời giải:

Đặt $\frac{x}{a}=\frac{y}{b}=\frac{z}{c}=t$

$\Rightarrow x=at; y=bt; z=ct$. Ta có:

$(x+y+z)^2=(at+bt+ct)^2=t^2(a+b+c)^2=t^2(*)$

Mặt khác:

$x^2+y^2+z^2=(at)^2+(bt)^2+(ct)^2=t^2(a^2+b^2+c^2)=t^2(**)$

Từ $(*); (**)\Rightarrow (x+y+z)^2=x^2+y^2+z^2$ (đpcm)

Khiêm Nguyễn Gia
Xem chi tiết
Lê Song Phương
8 tháng 12 2023 lúc 21:22

 Trước hết ta chứng minh BĐT sau: \(\dfrac{a^2}{x}+\dfrac{b^2}{y}\ge\dfrac{\left(a+b\right)^2}{x+y}\) (*) với \(a,b,x,y>0\). Thật vậy, (*) tương đương \(\dfrac{a^2y+b^2x}{xy}\ge\dfrac{a^2+2ab+b^2}{x+y}\)

 \(\Leftrightarrow a^2xy+a^2y^2+b^2x^2+b^2xy\ge2abxy+a^2xy+b^2xy\)

 \(\Leftrightarrow\left(ay-bx\right)^2\ge0\) (luôn đúng)

Vậy BĐT được chứng minh. ĐTXR \(\Leftrightarrow ay=bx\Leftrightarrow\dfrac{a}{x}=\dfrac{b}{y}\)

Áp dụng BĐT (*) liên tiếp, ta được:

 \(\dfrac{a^2}{x}+\dfrac{b^2}{y}+\dfrac{c^2}{z}\ge\dfrac{\left(a+b\right)^2}{x+y}+\dfrac{c^2}{z}\ge\dfrac{\left(a+b+c\right)^2}{x+y+z}\)

ĐTXR \(\Leftrightarrow\dfrac{a}{x}=\dfrac{b}{y}=\dfrac{c}{z}\)

Ta có đpcm.

2K9-(✎﹏ ΔΠGΣLS ΩҒ DΣΔTH...
Xem chi tiết
Nguyễn Việt Lâm
27 tháng 12 2022 lúc 19:48

1.

Áp dụng BĐT Cauchy-Schwarz:

\(\dfrac{a}{2a+a+b+c}=\dfrac{a}{25}.\dfrac{\left(2+3\right)^2}{2a+a+b+c}\le\dfrac{a}{25}\left(\dfrac{2^2}{2a}+\dfrac{3^2}{a+b+c}\right)=\dfrac{2}{25}+\dfrac{9}{25}.\dfrac{a}{a+b+c}\)

Tương tự:

\(\dfrac{b}{3b+a+c}\le\dfrac{2}{25}+\dfrac{9}{25}.\dfrac{b}{a+b+c}\)

\(\dfrac{c}{a+b+3c}\le\dfrac{2}{25}+\dfrac{9}{25}.\dfrac{c}{a+b+c}\)

Cộng vế:

\(VT\le\dfrac{6}{25}+\dfrac{9}{25}.\dfrac{a+b+c}{a+b+c}=\dfrac{3}{5}\)

Dấu "=" xảy ra khi \(a=b=c\)

Nguyễn Việt Lâm
27 tháng 12 2022 lúc 19:52

2.

Đặt \(\dfrac{x}{x-1}=a;\dfrac{y}{y-1}=b;\dfrac{z}{z-1}=c\)

Ta có: \(\dfrac{x}{x-1}=a\Rightarrow x=ax-a\Rightarrow a=x\left(a-1\right)\Rightarrow x=\dfrac{a}{a-1}\)

Tương tự ta có: \(y=\dfrac{b}{b-1}\) ; \(z=\dfrac{c}{c-1}\)

Biến đổi giả thiết:

\(xyz=1\Rightarrow\dfrac{abc}{\left(a-1\right)\left(b-1\right)\left(c-1\right)}=1\)

\(\Rightarrow abc=\left(a-1\right)\left(b-1\right)\left(c-1\right)\)

\(\Rightarrow ab+bc+ca=a+b+c-1\)

BĐT cần chứng minh trở thành:

\(a^2+b^2+c^2\ge1\)

\(\Leftrightarrow\left(a+b+c\right)^2-2\left(ab+bc+ca\right)\ge1\)

\(\Leftrightarrow\left(a+b+c\right)^2-2\left(a+b+c-1\right)\ge1\)

\(\Leftrightarrow\left(a+b+c-1\right)^2\ge0\) (luôn đúng)

Yoona
Xem chi tiết
Phương An
25 tháng 1 2017 lúc 16:22

\(\frac{1}{x}+\frac{1}{y}+\frac{1}{z}=0\)

\(\frac{yz}{xyz}+\frac{xz}{xyz}+\frac{xy}{xyz}=0\)

\(\frac{yz+xz+xy}{xyz}=0\)

yz + xz + xy = 0

\(\left(x+y+z\right)^2=x^2+y^2+z^2+2xy+2xz+2yz=x^2+y^2+z^2+2\times\left(xy+xz+yz\right)=x^2+y^2+z^2+2\times0=x^2+y^2+z^2\left(\text{đ}pcm\right)\)

Nguyen Bao Linh
25 tháng 1 2017 lúc 17:40

a) Từ giả thiết suy ra: xy + yz + zx = 0

Do đó:

\(\left(x+y+z\right)^2=x^2+y^2+z^2+2\left(xy+yz+zx\right)=x^2+y^2+z^2\)

b) Đặt \(\frac{1}{a-b}=x\); \(\frac{1}{b-c}=y\); \(\frac{1}{c-a}=z\)

Ta có: \(\frac{1}{x}+\frac{1}{y}+\frac{1}{z}=a-b+b-c+c-a=0\)

Theo câu a ta có: \(x^2+y^2+z^2=\left(x+y+z\right)^2\)

Suy ra điều phải chứng minh

Kuro Kazuya
25 tháng 1 2017 lúc 17:59

a)

\(\frac{1}{x}+\frac{1}{y}+\frac{1}{z}=0\)

\(\Rightarrow\frac{xy+yz+xz}{xyz}=0\)

\(\Rightarrow xy+yz+xz=0\)

\(x^2+y^2+z^2=\left(x+y+z\right)^2\)

\(\Rightarrow x^2+y^2+z^2=x^2+y^2+z^2+2xy+2yz+2xz\)

\(\Rightarrow x^2+y^2+z^2=x^2+y^2+z^2+2\left(xy+yz+xz\right)\)

Do \(xy+yz+xz=0\)

\(\Rightarrow x^2+y^2+z^2=x^2+y^2+z^2\) ( đpcm )

b)

\(\frac{1}{\left(a-b\right)^2}+\frac{1}{\left(b-c\right)^2}+\frac{1}{\left(c-a\right)^2}=\left(\frac{1}{a-b}+\frac{1}{b-c}+\frac{1}{c-a}\right)^2\)

\(\frac{1}{\left(a-b\right)^2}+\frac{1}{\left(b-c\right)^2}+\frac{1}{\left(c-a\right)^2}=\frac{1}{\left(a-b\right)^2}+\frac{1}{\left(b-c\right)^2}+\frac{1}{\left(c-a\right)^2}+\frac{2}{\left(a-b\right)\left(b-c\right)}+\frac{2}{\left(b-c\right)\left(c-a\right)}+\frac{2}{\left(a-b\right)\left(c-a\right)}\)

\(\Rightarrow\frac{2}{\left(a-b\right)\left(b-c\right)}+\frac{2}{\left(b-c\right)\left(c-a\right)}+\frac{2}{\left(a-b\right)\left(c-a\right)}=0\)

\(\Rightarrow2\left(\frac{1}{\left(a-b\right)\left(b-c\right)}+\frac{1}{\left(b-c\right)\left(c-a\right)}+\frac{1}{\left(a-b\right)\left(c-a\right)}\right)=0\)

\(\Rightarrow\frac{1}{\left(a-b\right)\left(b-c\right)}+\frac{1}{\left(b-c\right)\left(c-a\right)}+\frac{1}{\left(a-b\right)\left(c-a\right)}=0\)

\(\Rightarrow\frac{\left(c-a\right)^2\left(b-c\right)\left(a-b\right)+\left(a-b\right)^2\left(b-c\right)\left(c-a\right)+\left(b-c\right)^2\left(a-b\right)\left(c-a\right)}{\left(a-b\right)^2\left(b-c\right)^2\left(c-a\right)^2}=0\)

\(\Rightarrow\frac{\left(c-a\right)\left(b-c\right)\left(a-b\right)\left[\left(a-b\right)+\left(b-c\right)+\left(c-a\right)\right]}{\left(a-b\right)^2\left(b-c\right)^2\left(c-a\right)^2}=0\)

\(\Rightarrow\frac{\left(c-a\right)\left(b-c\right)\left(a-b\right)\left[\left(-a+a\right)+\left(-b+b\right)+\left(-c+c\right)\right]}{\left(a-b\right)^2\left(b-c\right)^2\left(c-a\right)^2}=0\)

\(\Rightarrow\frac{\left(c-a\right)\left(b-c\right)\left(a-b\right).0}{\left(a-b\right)^2\left(b-c\right)^2\left(c-a\right)^2}=0\)

\(\Rightarrow0=0\) ( đpcm )

Lê Thanh Quang
Xem chi tiết
Hoàng
Xem chi tiết
Manaka Mukaido
Xem chi tiết
Nguyễn Linh Chi
Xem chi tiết
tth_new
12 tháng 11 2019 lúc 19:22

1/ Không mất tính tổng quát, giả sử \(a\ge b\ge c\text{ và }x\ge y\ge z\)

Ta sẽ chứng minh \(ax+by+cz\ge\frac{1}{3}\left(a+b+c\right)\left(x+y+z\right)\)(Thấy giông giống BĐT Chebyshev nhưng không biết có phải không nên ko dám áp dụng, chứng minh cho chắc:D)

\(\Leftrightarrow3ax+3by+3cz\ge\left(a+b+c\right)\left(x+y+z\right)\)

\(\Leftrightarrow2\left(ax+by+cz\right)\ge a\left(y+z\right)+b\left(z+x\right)+c\left(x+y\right)\)

\(\Leftrightarrow\left(2x-y-z\right)a+\left(2y-z-x\right)b+\left(2z-x-y\right)c\ge0\)

\(\Leftrightarrow\left(2x-y-z\right)a-\left[\left(2x-y-z\right)+\left(2z-x-y\right)\right]b+\left(2z-x-y\right)c\ge0\)

\(\Leftrightarrow\left(2x-y-z\right)\left(a-b\right)+\left(2z-x-y\right)\left(c-b\right)\ge0\)

\(\Leftrightarrow\left(x-y\right)\left(a-b\right)+\left(x-z\right)\left(a-c\right)+\left(y-z\right)\left(b-c\right)\ge0\) (Đúng do giả sử)

Như vậy: \(VT\ge\frac{1}{3}\left(a+b+c\right)\left(x+y+z\right)+\sqrt{\left(a^2+b^2+c^2\right)\left(x^2+y^2+z^2\right)}\)

\(\ge\frac{1}{3}\left(a+b+c\right)\left(x+y+z\right)+\sqrt{\frac{\left(a+b+c\right)^2\left(x+y+z\right)^2}{9}}=\frac{2}{3}\left(a+b+c\right)\left(x+y+z\right)=VP\)

Ta có đpcm.

Is that true? Em không chắc ở cái bổ đề ban đầu, khi biến đổi có thể làm lộn, nhưng em lại ngại làm kỹ nên em đã làm tắt:v

Khách vãng lai đã xóa
Thắng Nguyễn
13 tháng 11 2019 lúc 13:01

Bài 1 nếu tự nhiên ép \(x\ge y\ge z \) đồng thời\(a\ge b \ge c\) thì lời giải rất vô duyên. Có thể làm cách khá như sau

Nếu đặt \(t=\sqrt{\frac{x^2+y^2+z^2}{a^2+b^2+c^2}}\) và giả sử \(\left(x,y,z\right)=\left(tp,tq,tr\right)\) thì ta có \(a^2+b^2+c^2=p^2+q^2+r^2\)

Khi đó cần cm \(ap+bq+cr+a^2+b^2+c^2\ge\frac{2}{3}\left(a+b+c\right)\left(p+q+r\right)\)

\(\Leftrightarrow\frac{4}{3}\left(a+b+c\right)\left(p+q+r\right)\le\left(a+p\right)^2+\left(b+q\right)^2+\left(c+r\right)^2\left(\text{*}\right)\)

Dùng bdt \(ab\le\frac{\left(a+b\right)^2}{4}\) và \(\frac{\left(a+b+c\right)^2}{3}\le a^2+b^2+c^2\) ta có:

\(VT\left(\text{*}\right)\le\frac{\left(a+b+c+p+q+r\right)^2}{3}\le\left(a+p\right)^2+\left(b+q\right)^2+\left(c+r\right)^2=VP\left(\text{*}\right)\)

Khách vãng lai đã xóa
tth_new
12 tháng 11 2019 lúc 19:12

Thâm à nha:) Bài 2 là IMO 2001, em đã nêu đáp án tại đây: Câu hỏi của IMO 2001 - Toán lớp 9 - Học toán với OnlineMath

Khách vãng lai đã xóa
Lê Thanh Hân
Xem chi tiết